LSAT and Law School Admissions Forum

Get expert LSAT preparation and law school admissions advice from PowerScore Test Preparation.

User avatar
 Dave Killoran
PowerScore Staff
  • PowerScore Staff
  • Posts: 5852
  • Joined: Mar 25, 2011
|
#77962
Complete Question Explanation
(The complete setup for this game can be found here: lsat/viewtopic.php?f=370&p=92208)

The correct answer choice is (E).

This question can be easily solved using proper List question technique.

Answer choice (A) is incorrect because it violates the second rule.

Answer choice (B) is incorrect because it violates the third rule.

Answer choice (C) is incorrect because it violates the fourth rule.

Answer choice (D) is incorrect because it violates the first rule.

Answer choice (E) is the correct answer.

Get the most out of your LSAT Prep Plus subscription.

Analyze and track your performance with our Testing and Analytics Package.